How to Verify Linear Approximations and Find Accurate Values for x?

  • Thread starter Thread starter benEE2018
  • Start date Start date
  • Tags Tags
    Linear
Click For Summary
To verify the linear approximation of 1/(1 + 3x)^3 at a = 0, the inequality (1/(1+3x)^3) - 0.1 < 1 - 9x < (1/(1+3x)^3) + 0.1 must be solved for x. The process involves solving each part of the inequality, which leads to cubic or quartic equations that may yield one to three real values. It is important to use suitable approximations for solving these non-trivial equations. Ultimately, identifying the relevant solutions and determining which intervals satisfy the inequality will provide the accurate values of x.
benEE2018
Messages
27
Reaction score
1

Homework Statement


Verify the given linear approximation at
a = 0.
Then determine the values of x for which the linear approximation is accurate to within 0.1. (Enter your answer using interval notation. Round your answers to three decimal places.)

1/(1 + 3x)^3 ≈ 1 − 9x

Homework Equations



I have no idea how to find the x values

The Attempt at a Solution



(1/(1+3x)^3)-0.1<1-9x<(1/(1+3x)^3)+0.1 is how my textbook sets up the problem and then all of a sudden they seem to compute the x values that is unknown to me. is it simply just solving the inequality for x (subtract 1 and then divide by -9 having x=0) thanks and any help will be appreciated.
 
Physics news on Phys.org
benEE2018 said:
is it simply just solving the inequality for x (subtract 1 and then divide by -9 having x=0)
Yes, it is just a matter of solving each inequality for x, but not setting x = 0 anywhere.
E.g. for (1+3x)-3-0.1<1-9x, solve the equation (1+3x)-3-0.1=1-9x to find the end of the interval. Of course, solving a cubic is non-trivial, so you should use a suitable approximation.
Solving each cubic will give you one or three real values. (Two is theoretically possible, but most unlikely.) You then need to figure out which ones are relevant and which side of them satisfies the inequality.
 
haruspex said:
Yes, it is just a matter of solving each inequality for x, but not setting x = 0 anywhere.
E.g. for (1+3x)-3-0.1<1-9x, solve the equation (1+3x)-3-0.1=1-9x to find the end of the interval. Of course, solving a cubic is non-trivial, so you should use a suitable approximation.
Solving each cubic will give you one or three real values. (Two is theoretically possible, but most unlikely.) You then need to figure out which ones are relevant and which side of them satisfies the inequality.

Actually, you get quartics, because
\frac{1}{(1+3x)^3}\pm 0.1 = 1-9x \Longrightarrow 1 \pm 0.1(1+3x)^3 = (1+3x)^3(1-9x)
 
Ray Vickson said:
Actually, you get quartics, because
\frac{1}{(1+3x)^3}\pm 0.1 = 1-9x \Longrightarrow 1 \pm 0.1(1+3x)^3 = (1+3x)^3(1-9x)
How true. Same advice, though.
 
Question: A clock's minute hand has length 4 and its hour hand has length 3. What is the distance between the tips at the moment when it is increasing most rapidly?(Putnam Exam Question) Answer: Making assumption that both the hands moves at constant angular velocities, the answer is ## \sqrt{7} .## But don't you think this assumption is somewhat doubtful and wrong?

Similar threads

  • · Replies 10 ·
Replies
10
Views
2K
Replies
2
Views
2K
Replies
5
Views
2K
  • · Replies 3 ·
Replies
3
Views
1K
  • · Replies 3 ·
Replies
3
Views
2K
  • · Replies 1 ·
Replies
1
Views
7K
  • · Replies 3 ·
Replies
3
Views
2K
  • · Replies 11 ·
Replies
11
Views
2K
  • · Replies 2 ·
Replies
2
Views
2K
  • · Replies 5 ·
Replies
5
Views
2K